24
$\begingroup$

Let $(A_i)_i$ be $n\times n$ matrices with entries in a field $K$ with characteristic $0$. We consider the equation (1) $f(X)=A_kX^k+\cdots+A_1X+A_0=0_n$ where $X\in\mathcal{M}_n(K)$ is unknown. Let $g = \det(\lambda^kA_k+\cdots+\lambda A_1+A_0)\in K[\lambda]$.

Question: is it true that, if $B$ is a solution of (1) ($f(B)=0_n$), then $g(B)=0$ ?

i) In 1884, Sylvester seemed to think that it is true, but, without proof (as often with him).

ii) This result is "proved" in: K. Kanwar. A generalization of the Cayley-Hamilton Theorem. Advances in PURE mathematics.2013. Yet, the proof is obviously false !! Recall that this journal has already been the subject of a scandal: http://boingboing.net/2012/10/19/math-journal-accepts-computer.html So the problem seems not settled.

$\endgroup$
12
  • 1
    $\begingroup$ I'm not following. How can you apply $g$ to a matrix? The domain of $g$ is the ground field. $\endgroup$
    – Todd Trimble
    Oct 25, 2013 at 18:00
  • 6
    $\begingroup$ @Todd: I think what was intended is that $\lambda^k A_k + ... + \lambda A_1 + A_0$ is a matrix with entries in $K[\lambda]$, so its determinant is an element of $K[\lambda]$, and we are applying this abstract polynomial to $B$. $\endgroup$ Oct 25, 2013 at 18:01
  • 1
    $\begingroup$ @QiaochuYuan Ah yes; thanks. Wonder if I should edit. $\endgroup$
    – Todd Trimble
    Oct 25, 2013 at 18:04
  • 4
    $\begingroup$ In applying g to B you need to decide on what side the matrix coefficients are... $\endgroup$ Oct 25, 2013 at 18:38
  • 2
    $\begingroup$ @suv: neither, it's the thing I mentioned in my comment above. Your formulation of the problem makes it trivially true if the $A_k$ are chosen to be on the left. $\endgroup$ Oct 25, 2013 at 19:03

6 Answers 6

16
$\begingroup$

Yes, this follows from known facts on matrix polynomials. There is a full characterization of spectral divisors of matrix polynomials in Gohberg, Lancaster, Rodman, Matrix Polynomials. They treat monic polynomials (i.e., $A_k=1$), but this is not a restriction, since one can make a Möbius transform to enforce it unless $g(\lambda)\equiv 0$.

They introduce so-called standard pairs of a matrix polynomial $A(\lambda)$, which are in some sense a generalization of the companion matrix. Then they prove (Thm 3.12) that if a polynomial $Q(\lambda)$ is a right divisor of $P(\lambda)$ then the standard pair of $Q$ is a restriction of that of $P$; translating it to companion matrices, it would mean that the companion matrix of $Q$ can be obtained as a restriction of that of $P$ to an invariant subspace.

We apply that to $\lambda I - B$ (for which the "companion matrix" is $B$ itself) and $P(\lambda)=\sum_{i=0}^k A_i \lambda^i$. In particular, this means that the Jordan structure of $B$ is a substructure of that of the companion matrix of $P(\lambda)$, hence the algebraic multiplicities of the eigenvalues of $B$ are less or equal than those of $P(\lambda)$, that is, the characteristic polynomial of $B$ is a divisor of your $g(\lambda)$.

Not sure how much of this is understandable -- I must admit that book has not the reputation for being an easy-to-read one in the community. A more self-contained approach to these topics is in: I. Gohberg, M.A. Kaashoek and P. Lancaster, General theory of regular matrix polynomials and band Toeplitz operators.

$\endgroup$
3
  • $\begingroup$ Incidentally, I searched for polynomial eigenvalue problems and ran into this reference just a minute ago :-) --- though somehow I'm still not sure if the claim holds without further qualification... $\endgroup$
    – Suvrit
    Oct 25, 2013 at 19:31
  • $\begingroup$ Ok, now I am convinced :-) $\endgroup$
    – Suvrit
    Oct 25, 2013 at 20:48
  • $\begingroup$ Federico, thanks for your first post ; Lancaster is a charming man but, unfortunately, his book (matrix polynomials) falls me hands. $\endgroup$
    – loup blanc
    Oct 26, 2013 at 17:07
14
$\begingroup$

Am I missing something or is Ilya Bogdanov's elimination of $A_0$ trick more or less a proof in itself?

Assume that $f\left(B\right) = 0_n$. Then, $0_n = f\left(B\right) = A_kB^k + A_{k-1}B^{k-1} + ... + A_0 = \sum\limits_{i=0}^k A_iB^i$. But

$\lambda^k A_k + \lambda^{k-1}A_{k-1} + ... + A_0 = \sum\limits_{i=0}^k \lambda^i A_i = \sum\limits_{i=0}^k \lambda^i A_i - \sum\limits_{i=0}^k A_iB^i$ (since $0 = \sum\limits_{i=0}^k A_iB^i$)

$= \sum\limits_{i=0}^k A_i \left(\lambda^i-B^i\right)$.

This polynomial is divisible by $\lambda-B$ on the right (because $\lambda^i-B^i$ is divisible by $\lambda-B$ for every $i$). Hence,

$\det\left(\lambda^k A_k + \lambda^{k-1}A_{k-1} + ... + A_0\right)$ is divisible by $\det\left(\lambda-B\right)$.

In other words, $g\left(\lambda\right)$ is divisible by $\det\left(\lambda-B\right)$ (since $\det\left(\lambda^k A_k + \lambda^{k-1}A_{k-1} + ... + A_0\right) = g\left(\lambda\right)$). Since $B$ is a root of the polynomial $\det\left(\lambda-B\right)$ (by the usual Cayley-Hamilton theorem), this yields that $B$ is a root of $g\left(\lambda\right)$, so that $g\left(B\right) = 0$, and we are done.

I agree with Yazdegerd III that the characteristic-$0$ assumption shouldn't be there. Even if my proof would use it, Ilya's observation that the result is a polynomial identity in the entries of $A_k$, $A_{k-1}$, ..., $A_1$ and $B$ should make it clear that it holds over any commutative ring.

$\endgroup$
0
6
$\begingroup$

To convince people that this is true, let me add a quick proof that works in the special case in which $B$ is diagonalizable with distinct eigenvalues.

By changing bases, we can assume it is in fact diagonal and equal to $\operatorname{diag}(\lambda_1,\lambda_2,\dots,\lambda_n)$. We have $0=\sum A_i B^i e_j=\sum A_i\lambda_j^i e_j$ for the $j$-th vector of the canonical basis $e_j$. Hence for each $\lambda_j$ the matrix $\sum A_i \lambda_j^i$ is singular, so the $\lambda_j$ are all roots of $g(\lambda)=\det p(\lambda)$. In particular, this means that $g(B)=0$.

More care is necessary if $B$ has multiple eigenvalues and Jordan blocks, of course.

$\endgroup$
4
  • 5
    $\begingroup$ Since the generic matrix has distinct eigenvalues, the general case follows. Actually, you may take the coefficients of $B$ and $A_1,\dots,A_k$ as independent variables, then define $A_0$ from $f(B)=0$ and check only this case, all others being its images. $\endgroup$ Oct 26, 2013 at 7:52
  • 1
    $\begingroup$ You are correct! This can be indeed turned into a complete proof with a genericity / continuity argument. Thanks for pointing it out, overall it's a much simpler solution than I expected at first. $\endgroup$ Oct 26, 2013 at 9:52
  • $\begingroup$ The result given by Federico is known. In fact, it is sufficient that $B$ is diagonalizable, because any eigenvalue of a solution is a root of $g$ and $g(B)=diag(g((\lambda_i))=0$. I thought that the previous result is not very interesting because, even if the roots of $g$ are simple, I do not see why the eigenvalues of any solution would be simple. But Ilya shows that I was wrong. About his comment, to fix the unknown and the constants, except one, as indeterminates, is a simple but great idea. Bravissimo... $\endgroup$
    – loup blanc
    Oct 26, 2013 at 17:09
  • $\begingroup$ Federico writes "This can be indeed turned into a complete proof with a genericity / continuity argument" and he is right. Indeed we must specialize $B$ and $A_1,\cdots,A_k$ in $K$ (assumed to be algebraically closed). Thus the polynomial $g$ is zero out of an algebraic set, that is in a Zariski-dense set....This works in charateristic $0$. Yazdegerd III asks for characteristic $>0$. Perhaps it is sufficient that $K$ is infinite ?? $\endgroup$
    – loup blanc
    Oct 26, 2013 at 17:21
5
$\begingroup$

Yes, this is precisely Theorem 4 in Chapter VIII, $\S$5 of F. R. Gantmacher, The Theory of Matrices, Vol. 1. The proof seems to be essentially that in darij grinberg's answer.

$\endgroup$
1
$\begingroup$

Thanks for Federico for pointing out my stupid matlab typo :-) the counterexample below is false, so am deleting it.


I must be missing something or doing something silly, because I think it is false. Here is an explicit counterexample:

Let $k=2$. Let $A_2=I$, $A_1=\left( \begin{array}{cc} -1 & -6 \\ 2 & -9 \end{array} \right)$, and $A_0=\left( \begin{array}{cc} 0 & 12 \\ -2 & 14 \end{array} \right)$. Verify that

$B^2+A_1B+A_0=0$ for $B=\left( \begin{array}{cc} 1 & 0 \\ 0 & 2 \end{array} \right)$.

Also, note that $\det(\lambda^2 I_2 + \lambda A_1+A_0) = 24-50 \lambda +35 \lambda ^2-10 \lambda ^3+\lambda ^4$. But,

\begin{equation*} 24-50B + 35B^2 -10B^3 + B^4 = \left( \begin{array}{cc} 0 & 24 \\ 24 & 0 \end{array} \right). \end{equation*}

$\endgroup$
3
  • 2
    $\begingroup$ $24-50B+35B^2-10B^3+B^4=0$ on my computer. I think you mistakenly ended your Matlab line with a "+24" rather than with a "+24*eye(2)". :) $\endgroup$ Oct 25, 2013 at 20:24
  • $\begingroup$ Indeed, Federico, I ended it with 24 instead of 24*eye!! aaarrgghh $\endgroup$
    – Suvrit
    Oct 25, 2013 at 20:46
  • $\begingroup$ I've made the same error several times --- We should really blame this error-prone Matlab's syntax here. $\endgroup$ Oct 25, 2013 at 20:49
0
$\begingroup$

Essentially darij grinberg's nice proof can be stated, without using cayley hamilton, and over any commutative ring K, briefly as follows: It suffices by the non commutative root factor theorem to show that f divides g from the right. But polynomials with matrix coefficients are isomorphic to matrices with polynomial entries, and the matrix corresponding to f does divide the matrix corresponding to g from the right (and from the left) by cramers rule. QED

$\endgroup$

Your Answer

By clicking “Post Your Answer”, you agree to our terms of service and acknowledge you have read our privacy policy.

Not the answer you're looking for? Browse other questions tagged or ask your own question.